Angular momentum operators and eigenfunctions

In summary, the conversation is about a problem involving converting wavefunctions into spherical polar coordinates and using them to find eigenvalues for the angular momentum operator. The first wavefunction yields an eigenvalue of 0, which is correct, while the second wavefunction does not match an eigenvalue and the justification for this is being discussed. The conversation also includes a helpful explanation for how to approach the problem and how to use the operator cross product.
  • #1
machofan
8
0

Homework Statement



yVt5RSm.png


Homework Equations


The Attempt at a Solution



I have tried inserting the first wavefunction into Lz which gets me 0 for the eigenvalue for the first wavefunction. Is this correct?

For the second wavefunction, I inserted it into Lz and this gets me -i*hbar*xAe^-r/a which is not equal to an eigenvalue*Ay*e-r/a, although I am not sure if this is the correct justification.

Any help is much appreciated, thank you.
 
Last edited:
Physics news on Phys.org
  • #2
Seems right to me.
 
  • #3
I'm also stuck on how to convert the wavefunctions into spherical polar coordinates, having tried to use the formula for spherical polar coordinates, I notice that the wavefunctions do not depend on the variables phi and theta, so I am unsure on how to approach the first problem.
 
  • #4
Hello fan, and welcome to PF.

You get 0 for ##\Psi_0## because it is symmetric under rotations around the z axis: there is no angular momentum. So I think you are right. It is an eigenfunction.

For ##\Psi_1## you basically want to show that ##{\bf L}_z \left( \Psi_1 \right ) = L_z\, \Psi_1 ## with ##{\bf L}_z## the operator and ##L_z## an eigenvalue (i.e. a number, possibly complex), has no solutions.

You may assume |A| is not equal to zero (because of the normalization) so the eigenvalue zero is already excluded. So if you multiply the right hand side from the left by ##\Psi_1^*## and integrate, that yields ##L_z##, which is nonzero.

Do the same thing with the lefthand side and show that it does give zero. That way you prove that the assumption that ##\Psi_1## is an eigenfunction leads to a false equation.
 
  • #5
machofan said:
I'm also stuck on how to convert the wavefunctions into spherical polar coordinates, having tried to use the formula for spherical polar coordinates, I notice that the wavefunctions do not depend on the variables phi and theta, so I am unsure on how to approach the first problem.
With ##z = r\cos\theta## you sure have a theta dependence!
 
  • #6
But the exercise wants you to write out the z-component of the operator cross product ##{\bf r} \times {\bf p}##, not the wave functions!
[edit] sorry, it wants you to do both. In the first part the Lz, later on the ##\Psi##
 
  • #7
BvU said:
Hello fan, and welcome to PF.

You get 0 for ##\Psi_0## because it is symmetric under rotations around the z axis: there is no angular momentum. So I think you are right. It is an eigenfunction.

For ##\Psi_1## you basically want to show that ##{\bf L}_z \left( \Psi_1 \right ) = L_z\, \Psi_1 ## with ##{\bf L}_z## the operator and ##L_z## an eigenvalue (i.e. a number, possibly complex), has no solutions.

You may assume |A| is not equal to zero (because of the normalization) so the eigenvalue zero is already excluded. So if you multiply the right hand side from the left by ##\Psi_1^*## and integrate, that yields ##L_z##, which is nonzero.

Do the same thing with the lefthand side and show that it does give zero. That way you prove that the assumption that ##\Psi_1## is an eigenfunction leads to a false equation.

That has really cleared things up, thank you!
 
Last edited:

What is Angular Momentum?

Angular momentum is a physical quantity that describes the rotational motion of an object around a fixed axis. It is a vector quantity that includes both the magnitude and direction of the rotational motion.

What are Angular Momentum Operators?

Angular momentum operators are mathematical operators that are used to describe angular momentum in quantum mechanics. They are represented by the letters Lx, Ly, and Lz and correspond to the three components of angular momentum along the x, y, and z axes.

What is the Commutation Relationship between Angular Momentum Operators?

The commutation relationship between angular momentum operators is given by [Lx, Ly] = iℏLz, [Ly, Lz] = iℏLx, and [Lz, Lx] = iℏLy. This means that the order in which the operators are applied matters and they do not commute with each other.

What are Eigenfunctions of Angular Momentum Operators?

Eigenfunctions of angular momentum operators are the solutions to the Schrödinger equation for a given Hamiltonian. They represent the stationary states of the system and have well-defined values for the angular momentum operator.

What is the Importance of Angular Momentum Operators and Eigenfunctions?

Angular momentum operators and eigenfunctions are important in quantum mechanics because they describe the rotational properties of particles and are used to solve many physical problems. They also play a crucial role in understanding the behavior of atoms and molecules and have applications in fields such as quantum computing and nuclear magnetic resonance imaging.

Similar threads

  • Advanced Physics Homework Help
Replies
5
Views
1K
  • Advanced Physics Homework Help
Replies
6
Views
2K
  • Advanced Physics Homework Help
Replies
1
Views
1K
  • Advanced Physics Homework Help
Replies
20
Views
2K
  • Quantum Physics
Replies
31
Views
2K
  • Advanced Physics Homework Help
Replies
14
Views
2K
Replies
1
Views
1K
  • Advanced Physics Homework Help
Replies
6
Views
2K
  • Advanced Physics Homework Help
Replies
1
Views
1K
Replies
3
Views
408
Back
Top